Understanding the expression for the probability that one r.v. is less than another

gadelke

New member
Joined
Jun 1, 2018
Messages
10
From reading pages like this, it appears that given two independent r.v.s with a support of
gif.latex
, the following holds:

(1)
gif.latex


What is the intuition behind this / explanation of this expression?
My initial thought about how to find the probability Pr[X<Y] would be to fix Y and find the probability that X is less than that point, and then integrate over Y - but I can't quite see how this fits in with the expression above.

It would be helpful to understand the intuition behind the expression so that I can adapt it to other situations.
Based on this, do the following expressions hold?

(2)
gif.latex


(3)
gif.latex
(where z is a constant)


Thank you for your help
 
Last edited:
From reading pages like this, it appears that given two independent r.v.s with a support of
gif.latex
, the following holds:

(1)
gif.latex


What is the intuition behind this / explanation of this expression?
My initial thought about how to find the probability Pr[X<Y] would be to fix Y and find the probability that X is less than that point, and then integrate over Y - but I can't quite see how this fits in with the expression above.

It seems to me that the integral means exactly what you say it should mean. Can you tell us what you think doesn't fit?

What I see is \(\displaystyle \displaystyle\int_0^\infty\int_0^y f_X(x)f_Y(y) dx dy = \int_0^\infty\left[\int_0^y f_X(x)dx \right]f_Y(y) dy = \int_0^\infty Pr[X<y]f_Y(y) dy\)
 
From reading pages like this, it appears that given two independent r.v.s with a support of
gif.latex
, the following holds:

(1)
gif.latex


What is the intuition behind this / explanation of this expression?
My initial thought about how to find the probability Pr[X<Y] would be to fix Y and find the probability that X is less than that point, and then integrate over Y - but I can't quite see how this fits in with the expression above.
But that is exactly what it does! First, note that \(\displaystyle f_Y(y)\) depends only upon y so can be taken out of the x integration. We can write that expression as
\(\displaystyle \int_0^\infty f_Y(y) \left(\int_0^y f_X(x)dx\right)dy\).

The integral inside the parentheses is the probability that x is less than a given y and the integral outside the parentheses extends that to all y.

It would be helpful to understand the intuition behind the expression so that I can adapt it to other situations.
Based on this, do the following expressions hold?

(2)
gif.latex


(3)
gif.latex
(where z is a constant)


Thank you for your help
 
But that is exactly what it does! First, note that \(\displaystyle f_Y(y)\) depends only upon y so can be taken out of the x integration. We can write that expression as
\(\displaystyle \int_0^\infty f_Y(y) \left(\int_0^y f_X(x)dx\right)dy\).

The integral inside the parentheses is the probability that x is less than a given y and the integral outside the parentheses extends that to all y.

Thank you so much for your help!

Resurrecting this thread more that 2 months later - realised it's much more relevant to my research now.
I just want to confirm how this works when there are restrictions on X and Y.

Suppose I have restrictions \(\displaystyle x \in (a, b) \) and \(\displaystyle y\in[0, b) \). Also suppose that the PDF of x is \(\displaystyle f_{X}(\cdot) \) and the PDF of y is \(\displaystyle f_{Y}(\cdot) \), with corresponding CDFs \(\displaystyle F_{X} \) and \(\displaystyle F_{Y} \). The two distributions are independent. (To clarify, the support of both distributions is still \(\displaystyle [0, \infty) \), these restrictions are just for the certain events as outlined below - so I want to make sure I don't include probabilities of things that I don't want)

Here are two scenarios that I am considering, followed by possible solutions based on the advice given in this thread so far:

1. \(\displaystyle \Pr(x<y) \) and also \(\displaystyle x \in (a, b) \) and \(\displaystyle y\in[0, b) \)

Solution 1.1:

\(\displaystyle \displaystyle{\Pr(x<y \cap x \in (a, b) \cap y\in[0, b)) = \int_{0}^{b} \int_{a}^{y} f_{Y}(y) f_{X}(x) dx dy} \)

Solution 1.2:

\(\displaystyle \displaystyle{\Pr(x<y \cap x \in (a, b) \cap y\in[0, b) = \int_{a}^{b} \int_{a}^{y} f_{Y}(y) f_{X}(x) dx dy} \) - This solution comes from the fact that if x<y, then y must be at least a given the restrictions on x?

Solution 1.3:


\(\displaystyle \displaystyle{\Pr(x<y \cap x \in (a, b) \cap y\in[0, b) = \int_{0}^{\infty} \int_{0}^{y} f_{Y}(y) f_{X}(x) dx dy} \times f_{Y}(b) \times \left(f_{X}(b) - f_{X}(a) \right) \) - is this equivalent to either of the other two representations? This would be great to know. But I'm suspecting there may be some problems with independence




2. \(\displaystyle \Pr(x>y) \) and also \(\displaystyle x \in (a, b) \) and \(\displaystyle y\in[0, b) \)

Solution 2.1:
\(\displaystyle \displaystyle{\Pr(x>y \cap x \in (a, b) \cap y\in[0, b) ) = \int_{0}^{b} \int_{y}^{b} f_{Y}(y) f_{X}(x) dx dy} \) - This answer is problematic as it does not restrict x to be above a

Solution 2.2:
\(\displaystyle \displaystyle{\Pr(x>y \cap x \in (a, b) \cap y\in[0, b) ) = \int_{a}^{b} \int_{y}^{b} f_{Y}(y) f_{X}(x) dx dy} \) - I don't think this is correct either because its restricting y to be above a, when y can be between 0 and a.

How do I incorporate a and b into these formulations? Thanks again for your help freemathhelp community.
 
Last edited:
Top